[Rozgrzewka OM][MIX][Nierówności] Nierówności

Zadania z kółek matematycznych lub obozów przygotowujących do OM. Problemy z minionych olimpiad i konkursów matematycznych.
Regulamin forum
Wszystkie tematy znajdujące się w tym dziale powinny być tagowane tj. posiadać przedrostek postaci [Nierówności], [Planimetria], itp.. Temat może posiadać wiele różnych tagów. Nazwa tematu nie może składać się z samych tagów.
Awatar użytkownika
Premislav
Użytkownik
Użytkownik
Posty: 15685
Rejestracja: 17 sie 2012, o 13:12
Płeć: Mężczyzna
Lokalizacja: Warszawa
Podziękował: 195 razy
Pomógł: 5219 razy

Re: [Rozgrzewka OM][MIX][Nierówności] Nierówności

Post autor: Premislav »

Ukryta treść:    
Jak ktoś ogarnięty to sprawdzi i potwierdzi poprawność, to wrzucę nowe.
bosa_Nike
Użytkownik
Użytkownik
Posty: 1660
Rejestracja: 16 cze 2006, o 15:40
Płeć: Kobieta
Podziękował: 70 razy
Pomógł: 445 razy

Re: [Rozgrzewka OM][MIX][Nierówności] Nierówności

Post autor: bosa_Nike »

W.M. pisze:I wtedy ja, na dany znak,
Prężąc i śmiejąc się szampańsko...
To rozwiązanie jest ok. Nie znam wprawdzie firmówki, ale trudno mi jest wyobrazić sobie coś prostszego i fajniejszego niż zauważenie klasyka: \(\displaystyle{ (x+y-z)(y+z-x)(z+x-y)\le xyz}\).
Uwagi:    
Awatar użytkownika
Premislav
Użytkownik
Użytkownik
Posty: 15685
Rejestracja: 17 sie 2012, o 13:12
Płeć: Mężczyzna
Lokalizacja: Warszawa
Podziękował: 195 razy
Pomógł: 5219 razy

Re: [Rozgrzewka OM][MIX][Nierówności] Nierówności

Post autor: Premislav »

Dla rzeczywistych dodatnich \(\displaystyle{ a,b,c}\) spełniających warunek \(\displaystyle{ abc=1}\) proszę wykazać, że
\(\displaystyle{ \frac{1}{1+a+b}+\frac{1}{1+b+c}+\frac{1}{1+c+a}\le \frac{1}{2+a}+\frac{1}{2+b}+\frac{1}{2+c}}\).
bosa_Nike
Użytkownik
Użytkownik
Posty: 1660
Rejestracja: 16 cze 2006, o 15:40
Płeć: Kobieta
Podziękował: 70 razy
Pomógł: 445 razy

Re: [Rozgrzewka OM][MIX][Nierówności] Nierówności

Post autor: bosa_Nike »

Ukryta treść:    
W ramach (m.in.) samokrytyki oddaję kolejkę.

Dodano po 1 dniu 4 godzinach 19 minutach 13 sekundach:
...sobie.:roll: Dobrze, w takim razie może coś takiego:

Dodatnie liczby rzeczywiste \(\displaystyle{ a,b,c}\) spełniają warunek \(\displaystyle{ a+b+c=\frac{1}{a^2}+\frac{1}{b^2}+\frac{1}{c^2}.}\) Udowodnij, że $$2(a+b+c)\ge\sqrt[3]{7a^2b+1}+\sqrt[3]{7b^2c+1}+\sqrt[3]{7c^2a+1}.$$
Awatar użytkownika
Premislav
Użytkownik
Użytkownik
Posty: 15685
Rejestracja: 17 sie 2012, o 13:12
Płeć: Mężczyzna
Lokalizacja: Warszawa
Podziękował: 195 razy
Pomógł: 5219 razy

Re: [Rozgrzewka OM][MIX][Nierówności] Nierówności

Post autor: Premislav »

Ukryta treść:    
bosa_Nike
Użytkownik
Użytkownik
Posty: 1660
Rejestracja: 16 cze 2006, o 15:40
Płeć: Kobieta
Podziękował: 70 razy
Pomógł: 445 razy

Re: [Rozgrzewka OM][MIX][Nierówności] Nierówności

Post autor: bosa_Nike »

No właśnie. Zadanie pochodzi z zawodów indywidualnych MEMO 2013 i jest całkiem standardowe, ale statystyki miało zaskakująco niskie. Nikt go nie zmaksował, a z ogólnej liczby sześćdziesięciorga zawodników tylko pięciu uzyskało za nie niezerową notę, przy czym akurat Polacy wypadli tu dość korzystnie na tle innych nacji (3x6/8).
Awatar użytkownika
Premislav
Użytkownik
Użytkownik
Posty: 15685
Rejestracja: 17 sie 2012, o 13:12
Płeć: Mężczyzna
Lokalizacja: Warszawa
Podziękował: 195 razy
Pomógł: 5219 razy

Re: [Rozgrzewka OM][MIX][Nierówności] Nierówności

Post autor: Premislav »

Nowe:
dla liczb rzeczywistych \(\displaystyle{ a_{1}, \ a_{2}, \ a_{3}}\) spełniających warunki
\(\displaystyle{ a_{1}+a_{2}+a_{3}=S, \ \frac{a_{i}^{2}}{a_{i}-1}>S, \ i\in\left\{1,2,3\right\}, \ a_{1}>1, \ a_{2}>1, \ a _{3}>1 }\)
proszę udowodnić nierówność
\(\displaystyle{ \frac{1}{a_{1}+a_{2}}+\frac{1}{a_{2}+a_{3}}+\frac{1}{a_{3}+a_{1}}>1}\)
bosa_Nike
Użytkownik
Użytkownik
Posty: 1660
Rejestracja: 16 cze 2006, o 15:40
Płeć: Kobieta
Podziękował: 70 razy
Pomógł: 445 razy

Re: [Rozgrzewka OM][MIX][Nierówności] Nierówności

Post autor: bosa_Nike »

Ukryta treść:    
Udowodnij, że dla dowolnych rzeczywistych liczb \(\displaystyle{ x,y,z}\) prawdziwa jest nierówność $$5\left(x^4+y^4+z^4\right)+7\left(x^3y+y^3z+z^3x\right)\ge 0.$$
Awatar użytkownika
arek1357
Użytkownik
Użytkownik
Posty: 5703
Rejestracja: 6 gru 2006, o 09:18
Płeć: Mężczyzna
Lokalizacja: blisko
Podziękował: 129 razy
Pomógł: 524 razy

Re: [Rozgrzewka OM][MIX][Nierówności] Nierówności

Post autor: arek1357 »

Jak założymy, że wszystkie zmienne są dodatnie lub wszystkie ujemne to nierówność prawdziwa...

Jak założymy, że jedna zmienna jest ujemna jest to równoważne , że dwie zmienne są ujemne, w związku z tym zakładam , że:

\(\displaystyle{ x \le 0}\)

zapiszmy to tak:

\(\displaystyle{ x^3(5x-7y)+z^3(5z-7x)+5y^4-7y^3z \ge 0}\)

lub tak , żeby było ładniej:

\(\displaystyle{ |x|^3(5|x|+7y)+z^3(5z+7|x|)+5y^4 \ge 7y^3z }\)

Prawa strona nie zależy od \(\displaystyle{ x}\) a lewa jest najmniejsza wtedy gdy \(\displaystyle{ x=0 }\) , wystarczy więc udowodnić, że:

\(\displaystyle{ 5z^4+5y^4 \ge 7y^3z/:z^4}\)

\(\displaystyle{ 5+5t^4 \ge 7t^3 , t= \frac{y}{z} }\)

Wystarczy zbadać funkcję:

\(\displaystyle{ f(t)=5t^4-7t^3+5 , t \ge 0}\)

a tu link:

Kod: Zaznacz cały

https://www.wolframalpha.com/input/?i=f%28t%29%3D5t%5E4-7t%5E3%2B5


cnd...
bosa_Nike
Użytkownik
Użytkownik
Posty: 1660
Rejestracja: 16 cze 2006, o 15:40
Płeć: Kobieta
Podziękował: 70 razy
Pomógł: 445 razy

Re: [Rozgrzewka OM][MIX][Nierówności] Nierówności

Post autor: bosa_Nike »

Dwa pierwsze zdania są ok, ta droga z pewnością może prowadzić do rozwiązania. Jednak wydaje mi się, że później srogo pokuśkałeś znaki. Wygląda mi to tak, jakbyś miał w wyjściowej nierówności minus przed siódemką i z takiej postaci dalej przekształcał. Spróbujesz to wytłumaczyć albo naprawić?
Awatar użytkownika
arek1357
Użytkownik
Użytkownik
Posty: 5703
Rejestracja: 6 gru 2006, o 09:18
Płeć: Mężczyzna
Lokalizacja: blisko
Podziękował: 129 razy
Pomógł: 524 razy

Re: [Rozgrzewka OM][MIX][Nierówności] Nierówności

Post autor: arek1357 »

Jak są wszystkie \(\displaystyle{ x,y,z}\) ujemne to przed siódemką nie będzie minusa...

jak jest:

\(\displaystyle{ x \le 0 , y>0, z>0}\) - jedna ujemna

otrzymasz:

\(\displaystyle{ 5(x^4+y^4+z^4)+7(-x^3y+y^3z-z^3x) \ge 0}\)

jak masz:

\(\displaystyle{ x,y \le 0, z>0}\) - dwie ujemnne

Przypadek będzie analogiczny:


\(\displaystyle{ 5(x^4+y^4+z^4)+7(x^3y-y^3z-z^3x) \ge 0}\)

sprowadza się to do:

\(\displaystyle{ y^3(5y-7z)+z^3(5z-7x)+5x^4 \ge -7x^3y}\)

Wszystko po lewej i prawej dodatnie...

Więc się już nie rozpisywałem tylko dałem wszystko pod "jeden mianownik" stąd mój zapis:

(Wystarczy tylko dla.: \(\displaystyle{ x \le 0}\))

czyli:

\(\displaystyle{ \left[ x^3(5x-7y)+z^3(5z-7x)+5y^4\right] -7y^3z \ge 0}\)

Z każdego przypadku wyjdę

Jak widać wszystko wystarczy sprowadzić do rozpatrzenie jednego przypadku ...

Wszystko symetryczne więc po co miałem młócić słomę...
Ostatnio zmieniony 13 kwie 2020, o 14:55 przez Jan Kraszewski, łącznie zmieniany 1 raz.
Powód: Zasady nie zmieniły się: "po co" nadal piszemy osobno.
bosa_Nike
Użytkownik
Użytkownik
Posty: 1660
Rejestracja: 16 cze 2006, o 15:40
Płeć: Kobieta
Podziękował: 70 razy
Pomógł: 445 razy

Re: [Rozgrzewka OM][MIX][Nierówności] Nierówności

Post autor: bosa_Nike »

Ja w ogóle nie wątpię, że z każdego przypadku wyjdziesz. Mnie interesuje, jaka zmiana znaków zmiennych pozwala z nierówności
bosa_Nike pisze: 12 kwie 2020, o 05:11 \(\displaystyle{ 5\left(x^4+y^4+z^4\right)+7\left(x^3y+y^3z+z^3x\right)\ge 0}\)
otrzymać
arek1357 pisze: 13 kwie 2020, o 13:05 zapiszmy to tak:

\(\displaystyle{ x^3(5x-7y)+z^3(5z-7x)+5y^4-7y^3z \ge 0}\)
:?:

Dobra, mniejsza o to.
arek1357 pisze: 13 kwie 2020, o 14:34 jak jest:

\(\displaystyle{ x \le 0 , y>0, z>0}\) - jedna ujemna

otrzymasz:

\(\displaystyle{ 5(x^4+y^4+z^4)+7(-x^3y+y^3z-z^3x) \ge 0}\)
Ta postać jest dobra, ta wcześniej nie była. I tu już obie strony będą zależeć od \(\displaystyle{ x}\), jeżeli przeniesiesz tak, by każda ze stron zawierała tylko nieujemne składniki.
Awatar użytkownika
Premislav
Użytkownik
Użytkownik
Posty: 15685
Rejestracja: 17 sie 2012, o 13:12
Płeć: Mężczyzna
Lokalizacja: Warszawa
Podziękował: 195 razy
Pomógł: 5219 razy

Re: [Rozgrzewka OM][MIX][Nierówności] Nierówności

Post autor: Premislav »

To ja miałem nieco inny, ale trochę podobny pomysł.
Ukryta treść:    
Awatar użytkownika
arek1357
Użytkownik
Użytkownik
Posty: 5703
Rejestracja: 6 gru 2006, o 09:18
Płeć: Mężczyzna
Lokalizacja: blisko
Podziękował: 129 razy
Pomógł: 524 razy

Re: [Rozgrzewka OM][MIX][Nierówności] Nierówności

Post autor: arek1357 »

Ta postać jest dobra, ta wcześniej nie była. I tu już obie strony będą zależeć od x
, jeżeli przeniesiesz tak, by każda ze stron zawierała tylko nieujemne składniki.
To było tak oczywiste, że o tym nie pisałem...
P.Kraszewski !!! mam dyslekcję...
bosa_Nike
Użytkownik
Użytkownik
Posty: 1660
Rejestracja: 16 cze 2006, o 15:40
Płeć: Kobieta
Podziękował: 70 razy
Pomógł: 445 razy

Re: [Rozgrzewka OM][MIX][Nierówności] Nierówności

Post autor: bosa_Nike »

Napisałeś za to coś wręcz przeciwnego, co było nieuzasadnione, a dalsza część Twojego rozwiązania na tym się opierała.

@Premislav - Twoje rozwiązanie jest ok.

Uwagi:    
ODPOWIEDZ